Autor Tema: Técnicas para probar desigualdades

0 Usuarios y 1 Visitante están viendo este tema.

01 Abril, 2009, 09:55 am
Leído 20721 veces

Luis Fuentes

  • el_manco
  • Administrador
  • Mensajes: 56,142
  • País: es
  • Karma: +0/-0
Desigualdad entre la media artimética y geométrica:

Sean \( a_1,\ldots,a_n \) números reales no negativos entonces:

\( \underbrace{\dfrac{a_1+\ldots+a_n}{n}}_{\mbox{media artimética}}\geq \underbrace{\sqrt[n]{a_1\cdot \ldots \cdot a_n}}_{\mbox{media geométrica}} \)

La igualdad se da sólo si \( a_1=\ldots=a_n \).

Prueba:

Spoiler
Si \( a_1=\ldots=a_n \) entonces es claro que se tiene la igualdad.

En otro caso existen \( a_i\neq a_j \). Sustituyendo \( a_i,a_j \) por \( \dfrac{a_i+a_j}{2} \) tenemos una nueva serie de números con la misma media aritmética y estrictamente mayor geométrica ya que:

\( \dfrac{(a_i+a_j)}{2}\dfrac{(a_i+a_j)}{2}-a_ia_j=\dfrac{(a_i-aj)^2}{4}>0 \)

Reiterando el argumento llegamos a una serie de números iguales con la misma media aritmética, que coincide con su geométrica pero que es estrictamente mayor que la media geométrica inicial.

OJO. Tiene un error pendiente de corregir. Gracias Teón.
[cerrar]



Desigualdad de reordenación:

Sean conjuntos de números reales:

\(  a_1\leq a_2\leq\ldots\leq a_n \) (I)
\(  b_1\leq b_2\leq\ldots \leq b_n \)  (II)

si \( c_1,\ldots,c_n \) es cualquier reordenación de los números \( b_1,\ldots,b_n \) se verifica:

\( \displaystyle\sum_{i=1}^n{a_ib_{n+1-i}}\leq\displaystyle\sum_{i=1}^n{a_ic_i}\leq \displaystyle\sum_{i=1}^n{a_ib_i} \) (III).

 Además, si las desigualdades (I) y (II) son estrictas también lo son las de (III).

Prueba:

Spoiler
Basta ver que, si en una reordenación \( c_1,\ldots,c_n \) intercambiamos dos elementos \( c_p,c_q \) con \( c_p<c_q \), entonces:

\( \underbrace{a_1c_1+\ldots+a_pc_p+\ldots+a_qc_q+\ldots+a_nc_n}_{S}\geq \underbrace{a_1c_1+\ldots+a_pc_q+\ldots+a_qc_p+\ldots+a_nc_n}_T \)

Pero:

\( S-T=a_pc_p+a_qc_q-a_pc_q-a_qc_p=(a_p-a_q)(c_p-c_q)\geq 0 \).
[cerrar]



Desigualdad de reordenación II (versión dual):

Sean conjuntos de números reales positivos:

\(  0\leq a_1\leq a_2\leq\ldots\leq a_n \) (I)
\(  0\leq b_1\leq b_2\leq\ldots \leq b_n \) (II)

si \( c_1,\ldots,c_n \) es cualquier reordenación de los números \( b_1,\ldots,b_n \) se verifica:

\( \displaystyle\prod_{i=1}^n{a_i+b_{n+1-i}}\geq\displaystyle\prod_{i=1}^n{a_i+c_i}\geq \displaystyle\prod_{i=1}^n{a_i+b_i} \) (III).

 Además, si las desigualdades (I) y (II) son estrictas también lo son las de (III).

Prueba:

Spoiler
Basta ver que, si en una reordenación \( c_1,\ldots,c_n \) intercambiamos dos elementos \( c_p,c_q \) con \( c_p<c_q \), entonces:

\( \underbrace{(a_1+c_1)\ldots(a_p+c_p)\ldots(a_q+c_q)\ldots(a_n+c_n)}_{S}\geq \underbrace{(a_1+c_1)\ldots(a_p+c_q)\ldots(a_q+c_p)\ldots(a_n+c_n)}_T \)

Pero:

\( \dfrac{S}{T}=\dfrac{(a_p+c_p)(a_q+c_q)}{(a_p+c_q)(a_q+c_p)}\leq 1 \).

ya que:

\(  (a_p+c_p)(a_q+c_q)-(a_p+c_q)(a_q+c_p)=\ldots=(a_p-a_q)(c_q-c_p)\leq 0 \).
[cerrar]



Desigualdad de Jensen:

Sea \( f:[a,b]\longrightarrow{}R \) una función convexa, entonces si \( x_1,\ldots,x_n\in  [a,b] \) y \( 0\leq r_1,\ldots,r_n, r_1+\ldots+r_n=1 \) se verifica:

\( f(r_1x_1+r_2x_2+\ldots+r_nx_n)\leq r_1f(x_1)+r_2f(x_2)+\ldots+r_nf(x_n) \)

Prueba:

Spoiler
Por inducción:

 - Para \( n=2 \) es la definición de función convexa.

 - Suponemos cierto para \( n \) y lo probamos para \( n+1 \):

\(  f\left(\displaystyle\sum_{i=1}^{n+1}{}r_ix_i\right)=f\left((1-r_{n+1})\displaystyle\sum_{i=1}^{n}\dfrac{r_i}{1-r_{n+1}}x_i+r_{n+1}x_{n+1}\right)\leq  \)

 (por hipótesis de inducción para \( n=2 \))

\(  \leq (1-r_{n+1})f\left(\displaystyle\sum_{i=1}^{n}{}\dfrac{r_i}{1-r_{n+1}}x_i\right)+r_{n+1}f(x_{n+1})\leq \)

 (por hipótesis de inducción para el caso \( n \))

\( \leq \displaystyle\sum_{i=1}^n{}r_if(x_i)+r_{n+1}f(x_{n+1}). \)
[cerrar]



10 Junio, 2009, 12:57 pm
Respuesta #1

Teón

  • $$\Large \color{#5b61b3}\pi\,\pi\,\pi\,\pi\,\pi$$
  • Mensajes: 1,369
  • Karma: +0/-0
  • Sexo: Masculino
  • C:.J:.T:.
Hola El_Manco, tengo una duda en la demostración que haces de la desigualdad entre la media artimética y geométrica.
Donde dices:
Citar
Si\( a_1=\ldots=a_n \)  entonces es claro que se tiene la igualdad.

En otro caso existen \( a_i\neq a_j \) . Sustituyendo \( a_i,a_j \)
  por \( \dfrac{a_i+a_j}{2} \)  tenemos una nueva serie de números con la misma media aritmética y estrictamente mayor geométrica ya que:
\( \dfrac{(a_i+a_j)}{2}\dfrac{(a_i+a_j)}{2}-a_ia_j=\dfrac{(a_i-a_j)^2}{4}>0 \)


Reiterando el argumento llegamos a una serie de números iguales con la misma media aritmética, que coincide con su geométrica pero que es estrictamente mayor que la media geométrica inicial.

Voy a poner como ejemplo un caso con tres números:

\( a_1,a_2,a_3 \), los cuales supongo distintos.
Puedo hacer dos cualesquiera de ellos iguales con la misma media aritmética.

\( \displaystyle\frac{a_1+a_2}{2},\displaystyle\frac{a_1+a_2}{2},a_3 \) ahora me queda \( a_3 \) distinto, pero si reitero el método con \( a_3 \) y alguno de los dos números, vuelve a quedarme otro distinto.

Aquí propongo una demostración algo diferente, aunque un poco más larga:
Spoiler
Lema 1.

Si \( \displaystyle\prod_{i=1}^n a_i=1  \) donde \( a_1, \ldots ,a_n \in \mathbb{R}^+ \)

Entonces \( \sum_{i=1}^n a_i \geq n \) donde la igualdad se cumple cuando \( a_1=a_2=\ldots =a_n \) lo cual es evidente.

Demostraremos por inducción sobre n.

para \( \displaystyle {n=2, \; a_1a_2=1 \rightarrow a_1+a_2=a_1+ \frac{1}{a_1}=\frac{a_1^2+1}{a_1}\geq 2} \) ya que
\( a_1^2+1-2a_1=\left(a_1-1 \right)^2\geq 0 \).

Supongamos que para \( \prod_{i=1}^n a_i=1 \) se cumple \(  \sum_{i=1}^n a_i \geq n \).

Si se tiene  \( \prod_{i=1}^{n+1} a_i=1 \) donde no todos los \( a_i \) son iguales, luego deben haber \( a_j>1, \; a_k <1 \)

Sea \( a=a_ja_k \),  se tiene \( (a_j-1)(a_k-1)<0 \rightarrow a\leq a_k+a_j-1 \)

Si \( \mathcal{A}=\{1, \ldots , n\} -\{ j,k\} , \quad a \displaystyle\prod_{i \in \mathcal{A}}a_i=1 \) luego por hipótesis inductiva

\( n \leq a+\displaystyle\sum_{i \in \mathcal{A}}a_i<a_k+a_j-1+\displaystyle\sum_{i \in \mathcal{A}}a_i \)

De donde se tiene
 \( n+1<a_k+a_j+\displaystyle\sum_{i \in \mathcal{A}}a_i=\displaystyle\sum_{i=1}^{n+1} a_i \). Como queríamos demostrar.

Si hacemos \( \displaystyle {a_i=\frac{x_i}{\left( \prod_{j=1}^n x_j\right)^\frac{1}{n}} \; , \quad \prod_{i=1}^n a_i=1} \)

y por el lema 1.
\( \displaystyle{\sum_{i=1}^n a_i=\frac{\sum_{i=1}^n x_i}{\left( \prod_{j=1}^n x_j\right)^\frac{1}{n}}\geq n} \)

O lo que es igual
\( \displaystyle{\frac{\sum_{i=1}^n x_i}{n}\geq \left( \prod_{j=1}^n x_j\right)^\frac{1}{n}} \)
[cerrar]
Eram quod es, eris quod sum.

10 Junio, 2009, 01:12 pm
Respuesta #2

Luis Fuentes

  • el_manco
  • Administrador
  • Mensajes: 56,142
  • País: es
  • Karma: +0/-0
Hola

 Tienes razón. Esa demostración la lei recientemente en unos apuntes que pululan por internet y me parecio buena por corta. Pero cierto, lo que se construye es una sucesión infinta de términos cada vez más parecidos entre si.

 Luego la corrijo. ¡Muchas gracias por la observación!.

Saludos.

10 Junio, 2009, 01:33 pm
Respuesta #3

Teón

  • $$\Large \color{#5b61b3}\pi\,\pi\,\pi\,\pi\,\pi$$
  • Mensajes: 1,369
  • Karma: +0/-0
  • Sexo: Masculino
  • C:.J:.T:.
Está bien El_manco.

Aqui va otra demostración, un poco más corta,  basada en que  \( -log(x)\; sobre \; \mathbb{R}^+ \), es una función convexa y aprovechando la desigualdad de Jensen.

Spoiler
Dados \( x_1, \ldots ,x_n \in \mathbb{R}^+ \)
\( \displaystyle{-log \left(\sum_{i=1}^n \frac{x_i}{n} \right)\leq -\sum_{i=1}^n \frac{log \left(x_i\right)} {n} =-log \left(\prod_{i=1}^n x_i \right)^\frac{1}{n} } \)

De donde se tiene

\( \displaystyle{log \left(\sum_{i=1}^n \frac{x_i}{n} \right)\geq log \left(\prod_{i=1}^n x_i \right)^\frac{1}{n} } \)

Es decir

\( \displaystyle{\frac{\sum_{i=1}^n x_i}{n} \geq \left(\prod_{i=1}^n x_i \right)^\frac{1}{n} } \)
[cerrar]

Saludos.
Eram quod es, eris quod sum.

10 Junio, 2009, 10:43 pm
Respuesta #4

Héctor Manuel

  • Lathi
  • Mensajes: 3,701
  • País: mx
  • Karma: +0/-0
  • Sexo: Masculino
DESIGUALDAD DE CHEBYSHEV

Se desprende de la desigualdad del reaocomodo, por lo que las hipótesis son las mismas:

\( \displaystyle\frac{a_1b_{n-1}+a_2b_{n-2}+...+a_{n-1}b_2+a_nb_1}{n}\leq{\displaystyle\frac{a_1+...+a_n}{n}\cdot{\displaystyle\frac{b_1+...+b_n}{n}}}\leq{\displaystyle\frac{a_1b_1+...+a_nb_n}{n}} \)

La demostración se puede hacer análogamente a la demostración de la desigualdad del reacomodo

22 Enero, 2016, 10:11 pm
Respuesta #5

cartesiano

  • $$\Large \color{#6a84c0}\pi$$
  • Mensajes: 43
  • Karma: +0/-0
  • Sexo: Masculino
Está bien El_manco.

Aqui va otra demostración, un poco más corta,  basada en que  \( -log(x)\; sobre \; \mathbb{R}^+ \), es una función convexa y aprovechando la desigualdad de Jensen.

Spoiler
Dados \( x_1, \ldots ,x_n \in \mathbb{R}^+ \)
\( \displaystyle{-log \left(\sum_{i=1}^n \frac{x_i}{n} \right)\leq -\sum_{i=1}^n \frac{log \left(x_i\right)} {n} =-log \left(\prod_{i=1}^n x_i \right)^\frac{1}{n} } \)

De donde se tiene

\( \displaystyle{log \left(\sum_{i=1}^n \frac{x_i}{n} \right)\geq log \left(\prod_{i=1}^n x_i \right)^\frac{1}{n} } \)

Es decir

\( \displaystyle{\frac{\sum_{i=1}^n x_i}{n} \geq \left(\prod_{i=1}^n x_i \right)^\frac{1}{n} } \)
[cerrar]

Saludos.
Me pregunto cómo demostrar que la igualdad se produce si y solo si \( x_1=x_2=\ldots=x_n \)

Saludos,

Gracias

23 Enero, 2016, 10:02 am
Respuesta #6

Luis Fuentes

  • el_manco
  • Administrador
  • Mensajes: 56,142
  • País: es
  • Karma: +0/-0
Hola


Me pregunto cómo demostrar que la igualdad se produce si y solo si \( x_1=x_2=\ldots=x_n \)


En el propio teorema de Jensen si la función es estrictamente convexa se puede probar que la igualdad sólo se tiene cuando los \( x_i  \)son iguales entre si.

Saludls.

23 Enero, 2016, 12:21 pm
Respuesta #7

cartesiano

  • $$\Large \color{#6a84c0}\pi$$
  • Mensajes: 43
  • Karma: +0/-0
  • Sexo: Masculino
Gracias, el_manco.

A ver si lo he entendido.
Una función \( f(x) \) es estrictamente convexa si dados \( x \neq y  \) , junto con \( 0<t<1 \) tenemos que
\( f(tx+(1-t)y)<tf(x)+(1-t)f(y) \)
Si una función f es estrictamente convexa, entonces dados \( x_1,x_2,\ldots,x_n \) no todos iguales, \( \displaystyle\sum_{i=1}^n{t_i}=1 \) con \( t_i>0, \forall{i}=1,\ldots,n \) Entonces se cumple la desigualdad estricta
\( f\left(\displaystyle\sum_{i=1}^n{t_ix_i}\right)<\displaystyle\sum_{i=1}^n{t_if(x_i)} \)
Esto se puede probar por inducción, igual que la desigualdad de Jensen. Sean \( x_i,\quad i=1,2,\ldots,n+1 \) y los correspondientes \( t_i \) con \( \displaystyle\sum_{i=1}^{n+1}{t_i}=1 \)
Si \( x_1=x_2=\ldots=x_n\neq x_{n+1} \) es esencialmente aplicar la desigualdad en el caso n=2
Si los \( x_i, \; i=1,2,\ldots,x_n \) no son todos iguales, entonces se puede reproducir lo mismo que en la demostración de la desigualdad de Jensen. de este ,mismo hilo.
\( f\left(\displaystyle\sum_{i=1}^{n+1}{t_ix_i}\right)=f\left((1-t_{n+1})\sum_{i=1}^n{\displaystyle\frac{t_i}{1-t_{n+1}}x_i+t_{n+1}x_{n+1}}\right)\\\textsf{(como f es convexa)}
\le(1-t_{n+1})f\left(\displaystyle\sum_{i=1}^n{\displaystyle\frac{t_i}{1-t_{n+1}}x_i}\right)+t_{n+1}f(x_{n+1})< \textsf{( paso por inducción)}\\
<(1-t_{n+1})\displaystyle\sum_{i=1}^n{\displaystyle\frac{t_i}{1-t_{n+1}}f(x_i)}+t_{n+1}f(x_{n+1})=\displaystyle\sum_{i=1}^{n+1}{t_i f(x_i)} \)

Como colorario, si f es estrictamente convexa y

\( f\left(\displaystyle\sum_{i=1}^{n}{t_ix_i}\right)=\displaystyle\sum_{i=1}^{n}{t_i f(x_i)} \)

entonces, necesariamente \( x_1=x_2=\ldots =x_n \)

Y desigualdad \( \displaystyle\prod_{i=1}^{n}{x_i^{p_i}}\le \displaystyle\sum_{i=1}^n{p_ix_i} \) con \( \displaystyle\sum_{i=1}^n{p_i}=1, p_i>0 \), dándose la igualdad si y solo si \( x_1=x_2=\ldots=x_n \)
Ésta se puede demostrar a partir de que la función \( -\log x \) es estrictamente convexa.

Saludos a todos,   :)
Gracias.